LSAT and Law School Admissions Forum

Get expert LSAT preparation and law school admissions advice from PowerScore Test Preparation.

User avatar
 Dave Killoran
PowerScore Staff
  • PowerScore Staff
  • Posts: 5850
  • Joined: Mar 25, 2011
|
#47078
Complete Question Explanation
(The complete setup for this game can be found here: lsat/viewtopic.php?t=8685)

The correct answer choice is (B)

Answer choice (A) is incorrect because R sits in seat G2.

Answer choice (B) is the correct answer.

Answer choice (C) is incorrect because this answer forms a QS or SQ block, and thus neither Q nor S could sit in the same row as R, a violation of the fourth rule.

Answer choices (D) and (E) are incorrect because R sits in row G, whereas T and U sit in row H.

Get the most out of your LSAT Prep Plus subscription.

Analyze and track your performance with our Testing and Analytics Package.